silver (unfinished gist)
Some checks failed
Build latex and deploy / checkout (push) Failing after 15m15s

This commit is contained in:
Josia Pietsch 2024-02-14 02:06:55 +01:00
parent ebdf929dd7
commit deaf6a6bb6
Signed by: josia
GPG key ID: E70B571D66986A2D
6 changed files with 180 additions and 36 deletions

View file

@ -9,7 +9,7 @@ Fix an ordinal $\beta$.
We recursively define
\begin{IEEEeqnarray*}{rClClr}
\beta &+& 0 &\coloneqq & \beta\\
\beta &+& (\alpha + 1)&\coloneqq &(\beta + \alpha),\\
\beta &+& (\alpha + 1)&\coloneqq &(\beta + \alpha) + 1,\\
\beta &+& \lambda &\coloneqq & \sup_{\alpha < \lambda} \beta + \alpha &~ ~\text{for limit ordinals $\lambda$}
\end{IEEEeqnarray*}
(Recall that $\alpha + 1 = \alpha \cup \{\alpha\}$

View file

@ -100,12 +100,13 @@
\item Consider $F\colon \bigcup_{i \in I}(\kappa_i \times \{i\}) \to \bigtimes_{i \in I} \lambda_i$. Want: $F$ is not surjective.
\item Let $\xi_i$ minimal such that $\underbrace{F((\eta,i))(i)}_{\in \lambda_i} \neq \xi$
for all $\eta < \kappa_i$.
\item $(i \mapsto \eta_i) \not\in \ran(F)$.
\item $(i \mapsto \xi_i) \not\in \ran(F)$.
\end{itemize}
}
\end{proof}
\begin{corollary}
\label{cor:cfpow}
For infinite cardinals $\kappa$,
it is $\cf(2^{\kappa}) > \kappa$.
\end{corollary}
@ -155,7 +156,7 @@
The next goal is to show the following:
(However the method might be more interesting than the result)
\begin{theorem}[Silver]
\yalabel{Silver's Theorem}{Silver}{thm:silver1}
\yalabel{Silver's Theorem (case of $\aleph_{\omega_1}$)}{Silver ($\aleph_{\omega_{1}}$)}{thm:silver1}
If $2^{\aleph_\alpha} = \aleph_{\alpha + 1}$
for all $\alpha < \omega_1$,
then $2^{\aleph_{\omega_1}} = \aleph_{\omega_1 + 1}$.

View file

@ -151,7 +151,8 @@ We have shown (assuming \AxC to choose contained clubs):
is defined to be
\[
\diagi_{\beta < \alpha} A_{\beta} \coloneqq
\{\xi < \alpha : \xi \in \bigcap \{A_{\beta} : \beta < \xi\} \}.
\{\xi < \alpha : \xi \in \bigcap \{A_{\beta} : \beta < \xi\} \}
= \bigcap_{\beta < \alpha} ([0,\beta] \cup A_\beta)
\]
\end{definition}
\begin{lemma}

View file

@ -35,7 +35,7 @@
Suppose otherwise.
Then for every $\nu$ there exists a club $C_\nu$
such that $S_\nu \cap C_\nu = \emptyset$.\footnote{Here we use \AxC to choose the $C_\nu$ uniformly.}
Let $C = \diagi_{\nu < \alpha} C_\nu$.
Let $C = \diagi_{\nu < \kappa} C_\nu$.
By \yaref{lem:diagiclub} $C$ is a club.
So we may pick some $\alpha \in C \cap S$.
In particular $\alpha \in C_\nu$ for all $\nu < \alpha$.
@ -46,8 +46,8 @@
\begin{itemize}
\item For $\nu < \kappa$ set $S_\nu \coloneqq \{\alpha \in S : f(\alpha) = \nu\}$.
\item Suppose none of the $S_\nu$ is stationary,
i.e.~ $\forall \nu < \kappa.~\exists C_\nu \text{ club}.~C_\nu \cap S_\nu = \neq$.
\item $\diagi_{\nu < \alpha} C_\nu$ is club.
i.e.~ $\forall \nu < \kappa.~\exists C_\nu \text{ club}.~C_\nu \cap S_\nu = \emptyset$.
\item $\diagi_{\nu < \kappa} C_\nu$ is club.
\item Pick $\alpha \in C \cap S$.
But then $\forall \nu < \alpha.~\alpha \in C_\nu$, i.e.~$f(\alpha) \ge \alpha$.
\end{itemize}
@ -104,10 +104,9 @@ to be an elementary substructure of $V_\theta$.
then $X \prec V_{\theta}$.
\end{lemma}
% TODO ANKI-MARKER
Let's do a second proof of \yaref{thm:fodor}.
\begin{refproof}{thm:fodor}
\gist{%
Fix $\theta > \kappa$ and look at $V_{\theta}$.
Fix $S \subseteq \kappa$ stationary
@ -158,13 +157,13 @@ Let's do a second proof of \yaref{thm:fodor}.
Let us show that $C$ is unbounded in $\kappa$.
Let $\zeta < \kappa$.
Let us define a strictly increasing sequence
$ \langle \xi_n n < \omega \rangle$
$ \langle \xi_n : n < \omega \rangle$
a follows.
Set $\xi_0 \coloneqq \zeta$.
Suppose $\xi_n$ has been chosen.
Look at $X_{\xi_n} \cap \kappa$.
Since $|X_{\xi_n} \cap \kappa| < \kappa$,
$\sup (X_{\xi_n \cap \kappa}) < \kappa$.
$\sup (X_{\xi_n} \cap \kappa) < \kappa$.
Set $\xi_{n+1} \coloneqq \sup(X_{\xi_n} \cap \kappa) + 1$.
Set $\xi \coloneqq \sup_{n<\omega} \xi_n$.
Clearly $\zeta < \xi$.
@ -234,8 +233,51 @@ Let's do a second proof of \yaref{thm:fodor}.
\]
Hence there is some $\eta \in X_\alpha$ with $\eta \in D$.
This means that
$\xi < \underbrace{\eta}_{\in D} < \alpha$..
$\xi < \underbrace{\eta}_{\in D} < \alpha$.
\end{subproof}
}{%
\begin{itemize}
\item Fix $S \subseteq \kappa$ stationary, $f\colon S \to \kappa$ regressive.
\item Fix $\theta > \kappa$ look at $V_\theta$,
fix Skolem functions $f_\phi$ over $V_{\theta}$ for all $\phi$.
\item Define sequence of elementary substructures $\langle X_\xi, \xi < \kappa \rangle$:
\begin{itemize}
\item $X_0$ minimal st.~closed under $f_\phi$ and $S,f \in X$ (note: countable).
\item $X_{\xi+1}$ minimal st.~$X_{\xi} \subseteq X_{\xi+1}$, closed under $f_\phi$ and
$\min(\kappa \setminus X_\xi) \in X$. (note: $|X_\xi| = |X_{\xi + 1}|$)
\item $X_{\lambda} \coloneqq \bigcup_{\xi < \lambda} X_\xi$
(note: cardinality may increase)
\end{itemize}
\item Note: strictly increasing, $|X_\xi| < \kappa$, $\xi \subseteq X_\xi$.
\item $C\coloneqq \{\xi < \kappa: X_\xi \cap \kappa = \xi\}$ is club:
\begin{itemize}
\item clearly closed.
\item unbounded in $\kappa$:
\begin{itemize}
\item Take $\zeta < \kappa$, Define $\langle \xi_n : n < \omega \rangle$ by
$\xi_0 \coloneqq \zeta$, $\xi_{n+1} \coloneqq \sup(X_{\xi_n} \cap \kappa)$
($ < \kappa$), $\xi \coloneqq \sup \xi_n < \kappa$.
\item $X_\xi \cap \kappa = \xi$, i.e.~$\xi \in C$:
\begin{itemize}
\item $\eta < \xi \implies \exists n.~\eta < \xi_n \implies \eta \in \xi_n \subseteq X_{\xi_n} \subseteq X_\xi$ $\implies \xi \subseteq X_\xi \cap \kappa$.
\item $\eta \in X_\xi \cap \kappa \implies \exists n.~\eta \in X_{\xi_n} \implies
\eta < \xi_{n+1} < \xi \implies X_\xi \cap \kappa \subseteq \xi$.
\end{itemize}
\end{itemize}
\end{itemize}
\item Take $\alpha \in S \cap C$, $\nu \coloneqq f(\alpha) < \alpha$,
$T \coloneqq \{\xi \in S : f(\xi) = \nu\}$.
($T \in X_\alpha$, since it can be defined from $S, f, \nu$)
\item $T$ is stationary:
Suppose $D \cap T = \emptyset$ for a club $D$.
$X_\alpha \prec V_\theta \implies$ find such $D \in X_\alpha$.
\item Clearly $\alpha \in T$. Want ($\lightning$) $\alpha \in D$:
\begin{itemize}
\item Suffices $\sup (D \cap \alpha) = \alpha$. Let $\xi < \alpha$.
\item $D$ unbounded, so $V_\theta \models \exists \eta > \xi : \eta \in D$
\item $\implies X_\alpha \models \exists \eta > \xi . ~\eta \in D$
\item $\implies \exists \eta > \xi. ~\eta \in D \land \underbrace{\eta < \alpha}_{\in X_\alpha}$.
\end{itemize}
\end{itemize}
}
\end{refproof}

View file

@ -1,5 +1,8 @@
\lecture{16}{2023-12-11}{}
% TODO ANKI-MARKER
Recall \yaref{thm:fodor}.
\begin{question}
What happens if $S$ is nonstationary?
@ -8,6 +11,7 @@ Let $S \subseteq \kappa$ be nonstationary,
$\kappa$ uncounable and regular.
Then there is a club $C \subseteq \kappa$
with $C \cap S = \emptyset$.
\gist{%
Let us define $f\colon S \to \kappa$ in the following way:
If $\alpha \in S$ and $C \cap \alpha \neq \emptyset$,
@ -29,13 +33,18 @@ where $\gamma' = \min(C \setminus (\gamma + 1))$.
Thus for all $\gamma$,
there is only an interval of ordinals $\alpha \in S$
where $f(\alpha) = \gamma$.
}{%
Consider $S\setminus \{0\} \ni \alpha \mapsto \underbrace{\max(C \cap \alpha)}_{< \alpha}$
(here $\max (\emptyset) \coloneqq 0$).
}
\todo{Move this to the definition of filter}
\gist{%
Recall that $F \subseteq \cP(\kappa)$ is a filter if
$X,Y \in F \implies X \cap Y \in F$,
$X \in X, X \subseteq Y \subseteq \kappa \implies Y \in F$
and $\emptyset \not\in F, \kappa \in F$.
\todo{Move this to the definition of filter?}
}{}
\begin{definition}
A filter $F$ is an \vocab{ultrafilter}
iff for all $X \subseteq \kappa$
@ -69,7 +78,7 @@ So neither $S_0$ nor $S_1 \subseteq \kappa \setminus S_0$ contains a club.
For $\kappa < \aleph_1$
this argument does not work, since there is only
on cofinality.
one cofinality.
\begin{theorem}[Solovay]
\yalabel{Solovay's Theorem}{Solovay}{thm:solovay}
@ -94,9 +103,11 @@ on cofinality.
\begin{refproof}{thm:solovay}%
\gist{%
%\footnote{``This is one of the arguments where it is certainly
% worth it to look at it again''}
% TODO: Look at this again and think about it.
% TODO TODO TODO
We will only proof this for $\aleph_1$.
Fix $S \subseteq \aleph_1$ stationary.
@ -208,15 +219,54 @@ on cofinality.
\]
Then $\langle S_i : i < \omega_1 \rangle$ is
as desired.
}{%
(Proof for $\aleph_1$)
Fix $S \subseteq \aleph_1$ stationary.
$\alpha \in (0, \omega_1)$ is successor ordinal or limit with $\cf(\alpha) = \omega_1$.
\begin{itemize}
\item $S^\ast \coloneqq \{\alpha \in S \setminus \{0\} : \alpha \text{ limit ordinal}\}$.
Still stationary:
$C$ club $\implies D = \{\alpha \in C \setminus \{0\} : \alpha \text{ limit}\}$ club.
\item For all $\alpha \in S^\ast$ choose $\langle \gamma^{\alpha}_n : n < \omega \rangle$ cofinal.
\item $\exists n < \omega.~\forall \delta < \omega_1: \{\alpha \in S^\ast : \gamma^{\alpha}_n > \delta\}$
stationary:
\begin{itemize}
% TODO THINK!
% TODO TODO TODO
\item Otherwise $\forall n < \omega.~\exists \delta.~\{\alpha \in S^\ast : \gamma^{\alpha}_n > \delta\} $ nonstationary.
\item $\delta_n\coloneqq $ least such $\delta$,
$C_n$ club s.t.~$C_n \cap \{\alpha \in S^\ast : \gamma^{\alpha}_n > \delta_n\} = \emptyset$.
(i.e.~$\alpha \in S^\ast \cap C_n \implies \gamma_n^{\alpha} \le \delta_n$).
\item $\delta^\ast \coloneqq \sup_{n < \omega} \delta_n$, $C \coloneqq \bigcap_{n < \omega} C_n$
is club. $\alpha \in S^\ast \cap C \implies \forall n.~\gamma_n^\alpha \le \delta^\ast$.
\item $C' \coloneqq C \setminus (\delta^\ast + 1)$ still club.
\item Pick $\alpha \in S^\ast \cap C'$.
But $\gamma_n^{\alpha} < \delta^\ast$ is cofinal $\lightning$.
\end{itemize}
\item Consider $f\colon S^\ast \to \omega_1, \alpha \mapsto \gamma_n^{\alpha}$ (note: regressive)
\item Define $\langle \delta_i : i < \omega_1 \rangle$ strictly increasing:
\begin{itemize}
\item $\delta_0 \coloneqq 0$,
\item $\delta'_{i} \coloneqq (\sup_{j < i} \delta_i) + 1$,
$\{\alpha \in S^\ast : \gamma_n^\alpha > \delta'_i\}$
is stationary $\overset{\yaref{thm:fodor}}{\implies} $ $\exists$ stationary $T_i \subseteq S^\ast$
s.t.~$f''T_i = \{\delta_i\} $ constant.
\end{itemize}
\item $T_i$ are disjoint: $i \neq j \implies \underbrace{f''T_i}_{\{\delta_i\} } \cap \underbrace{f''T_j}_{\{\delta_j\}} = \emptyset$.
\item $S_0 \coloneqq T_0 \cup \left( S \setminus \bigcup_{j > 0} T_j \right)$, $S_i \coloneqq T_i$.
\end{itemize}
}
\end{refproof}
\gist{%
We now want to do another application of \yaref{thm:fodor}.
Recall that $2^{\kappa} > \kappa$, in fact $\cf(2^{\kappa}) > \kappa$
by \yaref{thm:koenig}.
by \yaref{thm:koenig}
(cf.~\yaref{cor:cfpow}).
Trivially, if $\kappa \le \lambda$ then $2^{\kappa} \le 2^{\lambda}$.
This is in some sense the only thing we can prove about successor cardinals.
However we can say something about singular cardinals:
}{}
\begin{theorem}[Silver]
\yalabel{Silver's Theorem}{Silver}{thm:silver}
@ -231,6 +281,7 @@ However we can say something about singular cardinals:
is the statement
that $2^{\lambda} = \lambda^+$ holds for all infinite cardinals $\lambda$,
\end{definition}
\gist{%
Recall that $\CH$ says that $2^{\aleph_0} = \aleph_1$.
So $\GCH \implies \CH$.
@ -239,3 +290,4 @@ then it is true at $\kappa$.
The proof of \yalabel{thm:silver} is quite elementary,
so we will do it now, but the statement can only be fully appreciated later.
}{}

View file

@ -1,18 +1,24 @@
\lecture{17}{2023-12-14}{Silver's Theorem}
We now want to prove \yaref{thm:silver}.
More generally, if $\kappa$ is a singular cardinal of uncountable cofinality
such that $2^{\lambda} = \lambda^+$ for all $\lambda < \kappa$,
then $2^{\kappa} = \kappa^+$.
% More generally, if $\kappa$ is a singular cardinal of uncountable cofinality
% such that $2^{\lambda} = \lambda^+$ for all $\lambda < \kappa$,
% then $2^{\kappa} = \kappa^+$.
\gist{%
\begin{remark}
The hypothesis of \yaref{thm:silver}
is consistent with $\ZFC$.
\end{remark}
}{}
We will only proof \yaref{thm:silver} in the special case that $\kappa = \aleph_{\omega_1}$.
The general proof differs only in notation.
We will only proof
\gist{%
\yaref{thm:silver} in the special case that $\kappa = \aleph_{\omega_1}$
(see \yaref{thm:silver1}).
The general proof differs only in notation.
}{\yaref{thm:silver1}.}
\gist{%
\begin{remark}
It is important that the cofinality is uncountable.
For example it is consistent
@ -20,8 +26,10 @@ The general proof differs only in notation.
$2^{\aleph_n} = \aleph_{n+1}$ for all $n < \omega$
but at the same time $2^{\aleph_{\omega}} = \aleph_{\omega + 2}$.
\end{remark}
}{}
\begin{refproof}{thm:silver}
\begin{refproof}{thm:silver1}
\gist{%
We need to count the number of $X \subseteq \aleph_{\omega_1}.$
Let us fix $\langle f_\lambda : \lambda < \kappa \text{ an infinite cardinal} \rangle$
such that $f_{\lambda}\colon \cP(\lambda) \to \lambda^+$
@ -203,18 +211,58 @@ The general proof differs only in notation.
The set
\[
\{Y \subseteq \aleph_{\omega_1} : \exists i < \aleph_{\omega_1 + 1} .~Y \le X_i\}
P \coloneqq \{Y \subseteq \aleph_{\omega_1} : \exists i < \aleph_{\omega_1 + 1} .~Y \le X_i\}
= \bigcup_{i < \aleph_{\omega_1 + 1}} \{Y \subseteq \aleph_{\omega_1} : Y \le X_i\}
\]
has size $\le \aleph_{\omega_1 + 1}$
(in fact the size is exactly $\aleph_{\omega_1 + 1}$).
But
\[
\{Y \subseteq \aleph_{\omega_1} : \exists i < \aleph_{\omega_1 + 1} .~Y \le X_i\} = \cP(\aleph_{\omega_1 + 1})
\]
because if $X \subseteq \aleph_{\omega_1 + 1}$
On the other hand
$P = \cP(\aleph_{\omega_1})$
because if $X \subseteq \aleph_{\omega_1}$
is such that $X \nleq X_i$ for all $i < \aleph_{\omega_1 + 1}$,
then $X_i \le X$ for all $i < \aleph_{\omega_1 + 1}$,
so such a set $X$ does not exist by \yaref{thm:silver:p:c3}.
}{Need to count $X \subseteq \aleph_{ \omega_{1}}$.
\begin{itemize}
\item Fix bijections $f_\lambda\colon 2^{\lambda} \to \lambda^+$
for all infinite cardinals $\lambda < \kappa$.
\item For $X \subseteq \aleph_{ \omega_1}$ define $f_X\colon \omega_1 \to \aleph_{ \omega_1},
\alpha \mapsto f_{\aleph_\alpha}(X \cap \aleph_\alpha)$.
\item (1) $X \neq Y \iff f_X \neq f_Y$:
\begin{itemize}
\item $X \neq Y \iff \exists \alpha.~X \cap \aleph_\alpha \neq Y \cap \aleph_\alpha \iff \exists \alpha.~f_X(\alpha) \neq f_Y(\alpha)$.
\end{itemize}
\item $X \le Y :\iff \{\alpha < \omega_1 : f_X(\alpha) \le f_Y(\alpha)\}$ stationary.
\item (2) $X\le Y \lor Y \le X$:
Suppose $X \not\le Y, Y \not\le X$. Choose witnessing clubs $C, D$.
$C \cap D$ is club, but then $f_X(\alpha) \le f_Y(\alpha)$ or
$f_X(\alpha) \ge f_Y(\alpha)$ for $\alpha \in C \cap D$.
\item (3) $X \subseteq \aleph_{ \omega_1}$, then $|\underbrace{\{Y \subseteq \aleph_{ \omega_1} : Y \le X\}}_{A}| \le \aleph_{ \omega_1}$
\begin{itemize}
\item Suppose $|A| \ge \aleph_{ \omega_1 + 1}$.
\item $S_Y \coloneqq \{\alpha : f_Y(\alpha) \le f_X(\alpha)\}$ stationary for all $Y \in A$.
$2^{\aleph_1} = \aleph_2 \implies$ at most $\aleph_2$ such $S_Y$.
\item If $\forall S \in \cP( \omega_1).~ |\underbrace{\{Y \in A : S_Y = S\}}_{A_S}| < \aleph_{ \omega_1 + 1}$,
then $|A| \le \aleph_2 \cdot <\aleph_{ \omega_1 + 1}$ $\lightning$ $\aleph_{ \omega_1 + 1}$ regular.
\item So $\exists S \in \omega_1.~|A_S| = \aleph_{ \omega_1 + 1 + 1}$.
% TODO TODO TODO hier weiter!
\end{itemize}
\item Define sequence $\langle X_i : i < \aleph_{ \omega_1 + 1} \rangle$
of subsets of $\aleph_{\omega_1}$:
\begin{itemize}
\item Consider $\{Y \subseteq \aleph_{ \omega_1} : \exists j < i.~Y \le X_j\}$
(cardinality $ \le \aleph_{ \omega_1}$),
Take $X_i \subseteq \aleph_{ \omega_1}$
such that $X_i \subseteq \aleph_{ \omega_1}$
such that $X_i \not\le X_j$ for all $j < i$.
\item $P \coloneqq \{Y \subseteq \aleph_{ \omega_1} : \exists i < \aleph_{ \omega_1 + 1}.~Y \le X_i\}$
\end{itemize}
\item $|P| \overset{\text{in fact } =}{\le} \aleph_{ \omega_1 + 1} \aleph_{ \omega_1} = \aleph_{ \omega_1 + 1}$ by (3).
\item $X \in \cP(\aleph_{ \omega_1}) \setminus M \implies \forall i < \aleph_{ \omega_1 + 1}.~X_i \le X$
$\lightning$ (3).
Thus $P = \cP(\aleph_{ \omega_1})$.
\end{itemize}
}
\end{refproof}